Practice Test 1 (2) Flashcards

1
Q

Which of the following organizations provides errors and omissions insurance for the real estate industry? (Select one)

Bank of Canada, Errors and Omissions Dept.

Canadian Mortgage and Housing Corporation (CMHC)

Canadian Real Estate Association (CREA)

Institute of Real Estate Management (IREM)

Real Estate Insurance Exchange (REIX)

Service Alberta

A

Real Estate Insurance Exchange (REIX)

How well did you know this?
1
Not at all
2
3
4
5
Perfectly
2
Q

Kenny is a licensed assistant for Balloon Realty. He is under investigation for misconduct. Kenny decides rather than go through the hearing process, he is just going to apply to withdraw from membership. Which of the following statements are TRUE, with regard to the impact of the withdrawal request? (Select ALL that apply)

The Industry Council must accept the withdrawal request; proceedings would automatically cease.

The Industry Council may reject the application; proceedings would continue. If accepted, proceedings would cease.

Withdrawal results in a 5-year ban from licensing in the industry.

Withdrawal results in a 10-year ban from licensing in the industry.

Withdrawal results in a lifetime ban from holding only the licence Kenny currently held in the industry.

Withdrawal results in a lifetime ban from holding any licence in the industry.

A

The Industry Council may reject the application; proceedings would continue. If accepted, proceedings would cease.

Withdrawal results in a lifetime ban from holding any licence in the industry.

How well did you know this?
1
Not at all
2
3
4
5
Perfectly
3
Q

True or False: You must first be hired by a brokerage to finalize the licensing process and complete the authorization.

A

True

How well did you know this?
1
Not at all
2
3
4
5
Perfectly
4
Q

Which of the following occurs when the demand for properties outweighs the availability of properties? (Select one)

Market economy

Market equilibrium

Market shortage

Market surplus

A

Market Shortage

How well did you know this?
1
Not at all
2
3
4
5
Perfectly
5
Q

True or False: Real property consists of both physical objects (real estate) and common law rights.

True

False

A

True

How well did you know this?
1
Not at all
2
3
4
5
Perfectly
6
Q

Real estate professionals dealing with this type of real estate should be aware of the usefulness of being able to recognize the capacity of a particular piece of land for carrying animal units without undue stress. Because maximum long-term production is governed by stocking rates to ensure the land is not abused, a licensee with this knowledge will add value to a transaction.

Which of the following real estate types is described in the above statement? (Select one)

Aquatic

Commercial

Residential

Rural

A

Rural

How well did you know this?
1
Not at all
2
3
4
5
Perfectly
7
Q

True or False: A brokerage is allowed to deposit funds received on behalf of clients into any brokerage account, as long as the deposit happens within two days.

True

False

A

False

How well did you know this?
1
Not at all
2
3
4
5
Perfectly
8
Q

Alberta’s real estate regulatory body is designed to accomplish which of the following? (Select ALL that apply)

Assist in the prevention of fraud

Increase the compensation packages of brokerage associates

Protect the consumer

Provide and maintain licensing for individuals meeting requirements of the Act and Rules

Provide advertising for top real estate brokerages

A

Assist in the prevention of fraud

Protect the consumer

Provide and maintain licensing for individuals meeting requirements of the Act and Rules

How well did you know this?
1
Not at all
2
3
4
5
Perfectly
9
Q

Which residential property type requires working with developers and builders, and may require you to accompany the buyer on their first visit to a show home to receive any compensation from the builder? (Select one)

Condominium Sales

Mobile Home Sales

New Residential Sales

Residential Sales

Recreational Property Sales

Rural Residential Sales

A

New Residential Sales

How well did you know this?
1
Not at all
2
3
4
5
Perfectly
10
Q

Which of the following are likely information sources that will enable a licensee to stay informed of market relevant information? (Select ALL that apply)

CMHC statistics

Dictionaries

Encyclopedias

MLS®

Newspapers to see latest interest rates and market conditions.

Statistics Canada (information on existing capital investment from both the private and public sectors)

Stock Market

Wikipedia

A

CMHC statistics

MLS®

Newspapers to see latest interest rates and market conditions.

Statistics Canada (information on existing capital investment from both the private and public sectors)

How well did you know this?
1
Not at all
2
3
4
5
Perfectly
11
Q

The Real Estate Act requires self-regulation of the real estate industry. Self-regulation means that the government, in this case the province of Alberta, allows those within the industry to use their specialized knowledge to regulate the industry.

True or False: In practice, this means Investigation and disciplinary action for real estate professionals is managed by RECA.

True

False

A

True

How well did you know this?
1
Not at all
2
3
4
5
Perfectly
12
Q

Bill and Linda have come to your Residential Real Estate brokerage and are looking at purchasing a large tract of land containing 40 lots, currently zoned and subdivided for residential development land, but is currently undeveloped. They are interested in buying the whole of the neighborhood and instead of subdividing it, they intend to just build two homes (one for themselves, and one for Linda’s aging mother), and use the rest of it as pasture to raise cattle and horses. You are a licensee specializing in residential real estate.

In this situation, which of the following statements is TRUE. (Select the BEST answer)

You can represent them as long as the zoning is residential property.

Bill and Linda should be represented by a licensee who has rural specialization, because Bill and Linda intend to use it as agricultural property.

Bill and Linda should be represented by a licensee who has commercial specialization, because Bill and Linda are purchasing more than four lots in a single transaction.

You should consult with the broker, as the person representing them likely needs specialization in multiple areas.

A

You should consult with the broker, as the person representing them likely needs specialization in multiple areas.

How well did you know this?
1
Not at all
2
3
4
5
Perfectly
13
Q

True or False: RECA automatically suspends a license during an investigation.

True

False

A

False

How well did you know this?
1
Not at all
2
3
4
5
Perfectly
14
Q

The Rules outline the management responsibilities of a broker. Select all of the following that are broker responsibilities. (Select ALL that apply)

Ensure proper management and control of documents or records related to licensing, registrations, and related regulatory requirements.

Ensure all documents related to trades are maintained or made available for the purposes of a RECA review.

Ensure the brokerage name is clearly indicated in all marketing and advertising.

Ensure all trust monies are properly accounted for.

Provide licensees and personnel of the brokerage with written policies and procedures.

Provide licensees with brokerage policies and procedures for their personal trades.

A

Ensure all documents related to trades are maintained or made available for the purposes of a RECA review.

Ensure the brokerage name is clearly indicated in all marketing and advertising.

Ensure all trust monies are properly accounted for.

Provide licensees and personnel of the brokerage with written policies and procedures.

Provide licensees with brokerage policies and procedures for their personal trades.

How well did you know this?
1
Not at all
2
3
4
5
Perfectly
15
Q

Regarding Supply and Demand:

True or False: Supply is the amount of a product that’s available for purchase. Demand is how much consumers want to buy the product.

True3

False

A

True

How well did you know this?
1
Not at all
2
3
4
5
Perfectly
16
Q

There are two ways for real estate professionals to connect with a brokerage: as an employee or an independent contractor.

Which of the two methods is under the direction and control of the brokerage representing the employer in all business dealings? (Select one)

Employee

Independent Contractor

Both Employee and Independent Contractor

None of the above

A

Employee

How well did you know this?
1
Not at all
2
3
4
5
Perfectly
17
Q

To be a broker, an individual must complete a set of requirements. Select the item below that is NOT a requirement to be a broker. (Select one)

All four practice courses (Residential, Commercial, Rural, and Property Management) so they are familiar with all aspects of trading in real estate.

At least two years of industry experience in the previous five years.

Broker pre-licensing education, which focuses on the regulatory responsibility of brokers.

Have a minimum of at least 3 associate brokers registered with the broker prior to opening the brokerage.

A

Have a minimum of at least 3 associate brokers registered with the broker prior to opening the brokerage.

How well did you know this?
1
Not at all
2
3
4
5
Perfectly
18
Q

Which residential property type is a type of ownership where individual owners own a specific unit, become part of a corporation, and the owners need to understand the implications of common property, fees, and reserve funds? (Select one)

Condominium Sales

Mobile Home Sales

New Residential Sales

Residential Sales

Recreational Property Sales

Rural Residential Sales

A

Condominium Sales

How well did you know this?
1
Not at all
2
3
4
5
Perfectly
19
Q

The RECA board is comprised of a board of directors. The board of directors is composed of seven people. Three are appointed by the Minister. The other four board members are elected from where? (Select one)

All seven are appointed by the Minister.

Each of the four industry councils elects one member.

Each of the four quadrants of Alberta elects one member.

The four largest Real Estate Associations elects one member.

The Public selects them at general elections to serve for four-year terms (rotating one each year).

A

Each of the four industry councils elects one member.

How well did you know this?
1
Not at all
2
3
4
5
Perfectly
20
Q

True or False: If at any time there is a shortage of money in an account in which money is held in trust, the brokerage shall deposit the brokerage’s own money into the account as soon as the amount of the shortage is determined.

True

False

A

True

How well did you know this?
1
Not at all
2
3
4
5
Perfectly
21
Q

Clark is living in Manitoba currently, but will be moving to Alberta as soon as he secures a home to live in. He will be paying in cash for the home (no mortgage). It is currently a “seller’s market” and properties are going fast. Clark has employed you to look into several properties he has found on the MLS®. You look at the properties and all of them have serious problems. In fact, Clark’s “favourite” home has a roof leak that is so serious it has caused flooding inside. While the current owner tells you it is now patched and “works fine,” you have doubts about the condition of the roof. You share these doubts with Clark via email and he tells you that he wants you to make a cash offer on the home immediately, and to notify the seller that he is waiving the condition to have a home inspection. You pick up the phone and call Clark. You again advise Clark that you think this is a bad idea, and provide reasons why. He thanks you, but insists that he wants you to make the offer.

Which of the following General Obligations represents why you will submit the offer on Clark’s behalf, even though you disagree? (Select one)

Act honestly.

Act in person.

Disclose information.

Exercise care and skill.

Maintain confidentiality.

Negotiate favourable terms.

Obey lawful instructions

A

Obey lawful instructions

How well did you know this?
1
Not at all
2
3
4
5
Perfectly
22
Q

Historically, it was assumed that every associate in a brokerage had equal access to the same clients and client information and therefore, under common law, it was acknowledged that agency relationship was always between the consumer and the brokerage. Regardless of the type of real estate (residential, rural, commercial, or property management), practice and brokerage models have now evolved enabling individual agents to maintain client confidentiality.

In Alberta, real estate brokerages may operate as one of two different types of brokerages. What are they? (Select one)

Crown law, and designated agency brokerages.

Designated agency, and common law brokerages.

Designated agency, and sole agency brokerages.

Implied agency, and designated agency brokerages.

Implied agency, and transaction agency brokerages.

Sole agency, and common law agency brokerages.

A

Designated agency, and common law brokerages.

How well did you know this?
1
Not at all
2
3
4
5
Perfectly
23
Q

Trust and confidence helps to ensure a client that you are doing everything you can to put their interests first. Which of the following helps to build trust or confidence between you and your client? (Select ALL that apply)

Being available when needed.

Being open and approachable.

Supplying their name to third-party advertisers as a surprise value to the customer.

Supporting the client’s best interests through the process of buying or selling a home.

Sharing another client’s humiliating experience to build rapport.

Taking time to answer their questions and support them with your expertise.

Talking about all the bad clients you’ve ever had, so they know you are open with your communication.

Giving best advice for the client, even when it is against the brokerage ir agents interests

A

Being available when needed.

Being open and approachable.

Supporting the client’s best interests through the process of buying or selling a home.

Taking time to answer their questions and support them with your expertise.

Taking time to answer their questions and support them with your expertise.

How well did you know this?
1
Not at all
2
3
4
5
Perfectly
24
Q

Floyd and Katherine are now empty nesters. They are looking to sell their large 5-bedroom home on a cul-de-sac and find a medium-sized property that has a primary bedroom in the basement. While they are looking for a smaller property, they aren’t necessarily looking for a cheaper one. They want all the modern amenities and upgrades available in higher end properties.

From the following list below, match the above description with the type of client. (Select one)

First-time buyer/seller.

Move-up buyer/seller.

Move-down buyer/seller.

Investment buyer/seller.

Elderly property owner.

A

Move-down buyer/seller.

How well did you know this?
1
Not at all
2
3
4
5
Perfectly
25
Q

Floyd and Katherine are now empty nesters. They are looking to sell their large 5-bedroom home on a cul-de-sac and find a medium-sized property that has a primary bedroom in the basement. While they are looking for a smaller property, they aren’t necessarily looking for a cheaper one. They want all the modern amenities and upgrades available in higher end properties.

From the following list below, match the above description with the type of client. (Select one)

First-time buyer/seller.

Move-up buyer/seller.

Move-down buyer/seller.

Investment buyer/seller.

Elderly property owner.

A

Move-down buyer/seller.

How well did you know this?
1
Not at all
2
3
4
5
Perfectly
26
Q

A real estate licensee is required to have the professional skill and capacity to know and address certain issues in a transaction before they arise. This is considered a “standard of care.” Therefore, real estate professionals should not wait for clients to ask specific questions or request specific services if they already know what would be required to prevent harm to client interests.

The standard of care to which real estate professionals are held includes informing the client of what? (Select ALL that apply)

Things they do not know to ask about.

The possible consequences of their lawful instructions to you.

The potential results of any decisions made.

Any costs to the trade that they may not have anticipated.

A

Things they do not know to ask about.

The possible consequences of their lawful instructions to you.

The potential results of any decisions made.

Any costs to the trade that they may not have anticipated.

How well did you know this?
1
Not at all
2
3
4
5
Perfectly
27
Q

Fraudulent misrepresentation occurs when a person knowingly makes false statements.

True or False: If misrepresentation is fraudulent, the wronged party can sue for damages caused.

TRUE

FALSE

A

True

How well did you know this?
1
Not at all
2
3
4
5
Perfectly
28
Q

Consumers and real estate professionals have working relationships that provide value to both parties. Real estate professionals are compensated in the form of monetary compensation paid to the brokerage. The consumer receives value in the form of services largely determined by what the consumer wants to get out of the relationship. From the below list, select all the items which apply to the value of the real estate relationship from the consumer’s standpoint. (Select ALL that apply)

Determining the appropriate value for a property on the open market.

Determining what action can be taken that will enhance the perceived value of their property.

Following the client’s legal instructions.

Explaining the impact of key economic indicators on the market and pricing.

Securing funding sources for down payments that do not need to be reported to the government or documented for insured loans.

Finding a property within their price range in the area they wish to buy.

Managing and creating all required legal documentation.

Signing documentation with their signature when they cannot be easily reached.

Being liable for the full price of a property if the deal falls through.

Managing the transaction process from start to finish

Listing a home on the MLS® system.

A

Determining the appropriate value for a property on the open market.

Determining what action can be taken that will enhance the perceived value of their property.

Following the client’s legal instructions.

Explaining the impact of key economic indicators on the market and pricing.

Finding a property within their price range in the area they wish to buy.

Managing and creating all required legal documentation.

Managing the transaction process from start to finish

Listing a home on the MLS® system.

How well did you know this?
1
Not at all
2
3
4
5
Perfectly
29
Q

True or False: Real estate professionals have a fiduciary obligation to customers.

TRUE

FALSE

A

False

How well did you know this?
1
Not at all
2
3
4
5
Perfectly
30
Q

A seller or buyer customer is an individual who has elected not to be represented by an agent in a transaction but instead represents themselves. If you represent the seller in an agency relationship, and the buyer is unrepresented (a customer), what should you discuss with the seller? (Select ALL that apply)

That you must act in the best interest of the customer.

That you must share all information given to you by the customer, with the seller, even if it is confidential (unless it were received while acting as the customer’s agent).

That you will not give the customer information or advice that is not in the seller’s best interests.

That you will use your judgement on the customer’s behalf and give them advice.

A

That you must share all information given to you by the customer, with the seller, even if it is confidential (unless it were received while acting as the customer’s agent).

That you will not give the customer information or advice that is not in the seller’s best interests.

How well did you know this?
1
Not at all
2
3
4
5
Perfectly
31
Q

Which of the following are other professionals that may be involved in a real estate transaction who are NOT licensed by RECA. (Select ALL that apply)

Licensed Condominium manager.

Photographers.

Licensed Property Manager.

Real estate brokerage administrative assistants.

Real estate lawyers.

Licensed Appraiser

Licensed Home Inspectors

Licensed Mortgage broker

Mortgage specialist from a bank or credit union

Residential Measurement service

A

Photographers.

Real estate brokerage administrative assistants.

Real estate lawyers.

Licensed Home Inspectors

Mortgage specialist from a bank or credit union

Residential Measurement service

How well did you know this?
1
Not at all
2
3
4
5
Perfectly
32
Q

Review each of the following actions and select those that demonstrate poor ethics and professional conduct? (Select ALL that apply)

Failure to meet the minimum required standard of conduct as defined by RECA.

Misrepresentation of facts, whether unknowingly, or intentionally.

Observing all policies and procedures in place at your brokerage.

Operating in contravention of the Real Estate Act Rules in any respect.

Acting as a Facilitator in a Transaction Brokerage capacity at a Common Law Agency Brokerage.

Declining to take on a client, due to personality incompatibility.

A

Failure to meet the minimum required standard of conduct as defined by RECA.

Misrepresentation of facts, whether unknowingly, or intentionally.

Operating in contravention of the Real Estate Act Rules in any respect.

How well did you know this?
1
Not at all
2
3
4
5
Perfectly
33
Q

True or False: Any instruction provided by the client in an agency relationship is protected. Even if unlawful, the licensee may not inform the local authorities.

TRUE

FALSE

A

False

How well did you know this?
1
Not at all
2
3
4
5
Perfectly
34
Q

Evaluate each of the following fiduciary obligations, and determine if it applies to a Sole Agency relationship with a SELLER. (Select ALL that apply)

Confidentiality.

Full accounting.

Full disclosure.

Hold funds in a private account for the buyer.

Obedience to lawful instructions.

Reasonable care and skill.

Signatory authorization for all real estate related transactions

Undivided loyalty.

A

Confidentiality.

Full accounting.

Full disclosure.

Obedience to lawful instructions.

Reasonable care and skill.

Undivided loyalty.

How well did you know this?
1
Not at all
2
3
4
5
Perfectly
35
Q

True or False: A Judicial Sale is forced by the courts as an act towards financial recovery for a third party.

TRUE

FALSE

A

True

How well did you know this?
1
Not at all
2
3
4
5
Perfectly
36
Q

Carter has come to Balloon Real Estate Brokerage to find an agent to sell his home. He works with Pradeep as his real estate agent, and signs a written service agreement (WSA). Pradeep explains that the brokerage operates in a way that each associate has their own clientele within the brokerage, and that they do not share information about clients between agents. The WSA also clearly explains that Pradeep is the only associate at the brokerage who will be able to assist Carter on behalf of the brokerage. Three days later, Francine comes into the same brokerage to buy Carter’s home. She meets Pradeep at the door who invites her in. Francine mentions the home, and Pradeep tells Francine that he needs to introduce another associate to Francine, as Carter has requested that Pradeep not represent both the buyer and seller in the transaction. Francine meets with Alayah who happily takes her on as a buyer client.

In the above Scenario, what is the brokerage model, and in what capacity are the clients being represented? (Select one)

Designated Agency and Sole Agency.

Designated Agency and Transaction Brokerage.

Common Law Agency and Sole Agency.

Common Law Agency and Transaction Brokerage.

Implied Agency and Self-Representation.

Self-Representation and Sole Agency.

A

Designated Agency and Sole Agency.

How well did you know this?
1
Not at all
2
3
4
5
Perfectly
37
Q

There are multiple types of authority in representation relationships. In this type of authority, a perfectly competent owner delegates signing authority with a notarized legal document to another person when they are unavailable, such as if they are travelling abroad and difficult to reach. (Select one)

Assignee.

Assignor.

Dower.

Executor.

Power of Attorney.

Probate.

Trustee.

A

Power of Attorney.

How well did you know this?
1
Not at all
2
3
4
5
Perfectly
38
Q

True or False: In order to facilitate each transaction fairly, a licensee in a transaction brokerage situation should provide copies of any documents or information requested by one side to the other side as well, so that each receives equal level of service.

TRUE

FALSE

A

True

How well did you know this?
1
Not at all
2
3
4
5
Perfectly
39
Q

The following list contains duties owed to clients in representation relationships and customers in non-representation relationships. Select the duties owed to customers in non-representation relationships (Select ALL that apply)

Giving them names of real estate service providers without recommending any specific service provider.

Giving them statistics and information on property, including comparable property information from listing services or other local databases.

Representing the client in a negotiation.

Presenting all offers and counter-offers to and from a customer, even if a seller may have accepted another purchase contract.

Providing a purchase contract document and other relevant forms, and helping them to complete them without advice.

Providing opinions of trending prices in the market for a specific area of town.

A

Giving them names of real estate service providers without recommending any specific service provider.

Giving them statistics and information on property, including comparable property information from listing services or other local databases.

Presenting all offers and counter-offers to and from a customer, even if a seller may have accepted another purchase contract.

Providing a purchase contract document and other relevant forms, and helping them to complete them without advice.

How well did you know this?
1
Not at all
2
3
4
5
Perfectly
40
Q

When purchasing a judicial sale property, in consideration of the serious risk of material changes to the state of the property between the offer acceptance date and the possession date, what strategy can be employed to protect the buyer? (Select one)

Hire a home inspector to do due diligence on the property prior to placing a bid, access is guaranteed by the courts.

Make it a condition of the bid to have the client act as “tenant” of the property during the acceptance and possession date, to enable the buyer to care for the property.

Order a pre-inspection certification, which is mandatory to be provided when requested of the courts.

Talk to an insurance broker and see what options are available for your buyer to purchase insurance to cover their financial interest in the home pre-possession.

A

Talk to an insurance broker and see what options are available for your buyer to purchase insurance to cover their financial interest in the home pre-possession.

How well did you know this?
1
Not at all
2
3
4
5
Perfectly
41
Q

True or False: The executor of an estate becomes the automatic property owner of the property to be disposed.

TRUE

FALSE

A

False

How well did you know this?
1
Not at all
2
3
4
5
Perfectly
42
Q

When using electronic advertising, including internet ads, which of the following is not an absolute requirement? (Select one)

Be true and accurate.

Clearly display the full name of the brokerage.

Disclose key information, including sources of information.

Use clearly marked disclaimers (hyperlinks) for additional information.

A

Use clearly marked disclaimers (hyperlinks) for additional information.

How well did you know this?
1
Not at all
2
3
4
5
Perfectly
43
Q

Which of the following are examples of fraud in real estate? (Select ALL that apply)

Answering the phone for a colleague to take a message.

Buyers who lend their name to the acquisition of property and mortgage application with no intention of becoming the owner other than on paper.

Pretending to be the property owner and leasing the property to a tenant.

Pretending to be the registered owner of a property to sell, mortgage, or otherwise encumber the property.

Verbally introducing yourself by your preferred nickname, instead of the name on your license.

A

Buyers who lend their name to the acquisition of property and mortgage application with no intention of becoming the owner other than on paper.

Pretending to be the property owner and leasing the property to a tenant.

Pretending to be the registered owner of a property to sell, mortgage, or otherwise encumber the property.

How well did you know this?
1
Not at all
2
3
4
5
Perfectly
44
Q

Failure to comply with the PCMLTFA may result in criminal or administrative monetary penalties. Criminal penalties can range from $500,000 to $2,000,000 dollars in fines and or imprisonment for up to five years.

In addition to criminal penalties, administrative monetary penalties can range up to $500,000 depending on the severity of the violation.

TRUE

FALSE

A

True

How well did you know this?
1
Not at all
2
3
4
5
Perfectly
45
Q

Proceeds of Crime (Money Laundering) and Terrorist Financing Act (PCMLTFA) differs from the Financial Transactions and Reports Analysis Centre of Canada (FINTRAC) in which of the following ways? (Select one)

PCMLTFA has an extensive website that provides numerous guidelines and notices for businesses that are mandated to use their system.

PCMLTFA is an independent agency, whereas FINTRAC is legislation.

PCMLTFA is legislation, whereas FINTRAC is an agency governed by the PCMLTFA.

There is no substantial difference, both are legislations that deal with money laundering.

A

PCMLTFA is legislation, whereas FINTRAC is an agency governed by the PCMLTFA.

How well did you know this?
1
Not at all
2
3
4
5
Perfectly
46
Q

Which of the PIPEDA’s Fair Information Principles is described as when a person gives permission to an organization to possess personal information? (Select one)

Accountability

Accuracy

Consent

Challenging Compliance

Identifying purposes

Individual Access

Limiting Collection

Limiting Use, Disclosure, Retention

Openness

Safeguards

A

Consent

How well did you know this?
1
Not at all
2
3
4
5
Perfectly
47
Q

To determine if the claim was false or misleading material, a court would consider whether an average consumer would likely be influenced into obtaining the product or service.

TRUE

FALSE

A

True

How well did you know this?
1
Not at all
2
3
4
5
Perfectly
48
Q

Which of the PIPEDA’s Fair Information Principles states that an organization must have policies and practices regarding how it manages personal information? (Select one)

Accountability

Accuracy

Consent

Challenging Compliance

Identifying purposes

Individual Access

Limiting Collection

Limiting Use, Disclosure, Retention

Openness

Safeguards

A

Identifying purposes

How well did you know this?
1
Not at all
2
3
4
5
Perfectly
49
Q

The Competition Act prohibits representations made to promote a product, service, or business interest that are false or misleading. Which of the following does this apply? (Select ALL that apply)

Marketing

Advertising

Social Media

Oral Claims

A

Marketing

Advertising

Social Media

Oral Claims

How well did you know this?
1
Not at all
2
3
4
5
Perfectly
50
Q

Real estate associates have a responsibility to keep client information protected. Which of the following are included in the individual real estate associate’s responsibilities? (Select ALL that apply)

Make a reasonable effort to ensure the information is accurate.

Notify the Privacy Commissioner of any incident involving loss, unauthorized access, or disclosure of personal information (breaches)where a reasonable person would consider there is a real risk of significant harm to an individual because of that breach.

Prepare and implement a compliance program consisting of appointing a compliance officer, maintaining current written compliance policies and procedures, training, and risk assessment.

Protect personal information by making sure they have reasonable security measures against risks.

A

Make a reasonable effort to ensure the information is accurate.

Notify the Privacy Commissioner of any incident involving loss, unauthorized access, or disclosure of personal information (breaches)where a reasonable person would consider there is a real risk of significant harm to an individual because of that breach.

Protect personal information by making sure they have reasonable security measures against risks.

How well did you know this?
1
Not at all
2
3
4
5
Perfectly
51
Q

As a licensee, you see different phrases for “personal information” that sound as though they mean the same thing. The three most common phrases are personal information, privacy, and confidential information. Which of the three phrases is information about an identifiable individual? (Select one)

Personal information

Privacy

Confidential information

A

Personal information

How well did you know this?
1
Not at all
2
3
4
5
Perfectly
52
Q

As a licensee, you see different phrases for “personal information” that sound as though they mean the same thing. The three most common phrases are personal information, privacy, and confidential information. Which of the three phrases is information about an identifiable individual? (Select one)

Personal information

Privacy

Confidential information

A

Personal information

How well did you know this?
1
Not at all
2
3
4
5
Perfectly
53
Q

To avoid making false or misleading statements, a real estate professional should always verify that their claim is true and accurate.

TRUE

FALSE

A

True

How well did you know this?
1
Not at all
2
3
4
5
Perfectly
54
Q

All of the following are examples of intentional false representations except which item? (Select the ONE that is not a false representation)

Advertising a certain commission rate but not disclosing that it is only for limited services.

Advertising where the description or photos of a property accurately show the property.

Showing lake front photos where the property is not lake front property to get additional interest

Advertised services are not included in the advertised commission.

Telling a buyer or real estate professional that there are competing bids when it is not true.

A

Advertising where the description or photos of a property accurately show the property.

How well did you know this?
1
Not at all
2
3
4
5
Perfectly
55
Q

During a real estate transaction, the buying associate must fill out a Fintrac ID on both the buyer and seller. If the seller refuses, the transaction cannot proceed.

TRUE

FALSE

A

False

How well did you know this?
1
Not at all
2
3
4
5
Perfectly
56
Q

Which of the following describes the purpose of the Environmental Protection and Enhancement Act? (Select one)

Enable the smaller government systems to create their own bylaws and grant approvals.

Grant ownership of mines and minerals associated with real property.

The legal framework upon which rights are granted by application to individuals or companies interested in harvesting the land’s mineral rich resources.

Support, protect, enhance, and ensure the sustainable use of our environment.

A

Support, protect, enhance, and ensure the sustainable use of our environment.

How well did you know this?
1
Not at all
2
3
4
5
Perfectly
57
Q

Which of the following groups have the right to have their personal information held as confidential and the right to choose what information to make available to others? (Select ALL that apply)

Client

Guest

Employee

Colleagues

A

Client

Guest

Employee

Colleagues

How well did you know this?
1
Not at all
2
3
4
5
Perfectly
58
Q

Obtaining consent is key when processing someone’s personal information.

Consent is when a person provides information to you, even when they do not know how you will utilize the information.

TRUE

FALSE

A

FALSE

How well did you know this?
1
Not at all
2
3
4
5
Perfectly
59
Q

As a licensee, you see different phrases for “personal information” that sound as though they mean the same thing. The three most common phrases are personal information, privacy, and confidential information. Which of the three phrases is the right of an individual to choose what information they make available to others, including an organization or government agency (Select one)

Personal information

Privacy

Confidential information

A

Privacy

How well did you know this?
1
Not at all
2
3
4
5
Perfectly
60
Q

Regarding Canada’s Anti-Spam Legislation (CASL), which of the following statements is NOT true. (Select one)

All electronic newsletters and marketing material need to have a clear “unsubscribe” capability.

Even if the client has given their permission to be sent electronic marketing, they have the right to change their mind and opt out.

Sending an email to request consent to send email is prohibited by CASL

It is a good practice and common brokerage policy to speak with your privacy officer or manager when embarking on a new marketing campaign.

Real estate professionals must have written permission for each call prior to the time they make a phone call to a client.

A

Real estate professionals must have written permission for each call prior to the time they make a phone call to a client.

How well did you know this?
1
Not at all
2
3
4
5
Perfectly
61
Q

Which of the PIPEDA’s Fair Information Principles indicates that a person has the right to know how the organization collects and protects their information, AND has the right to address any questions or concerns and understand the organization’s policies and procedures? (Select one)

Accountability

Accuracy

Consent

Challenging Compliance

Identifying Purposes

Individual Access

Limiting Collection

Limiting Use, Disclosure, Retention

Openness

Safegaurds

A

Accountability

62
Q

The Competition Act is legislation that is designed to ensure small and medium-size businesses have an equitable opportunity to participate in real estate transactions.

TRUE

FALSE

A

True

63
Q

Of the following personal safety best practices, which reduces personal risk by lowering the likelihood of loss due to theft? (Select one)

Be aware of other vehicles around you when you are getting in and out of your vehicle.

Do not get backed into corners or areas without an exit.

Make sure your cell phone’s battery is charged.

Make sure your purse, wallet and other valuables are tucked away out of sight or even in your vehicle trunk.

Never enter a basement ahead of someone else, effectively cutting yourself off from any potential exit.

A

Make sure your purse, wallet and other valuables are tucked away out of sight or even in your vehicle trunk.

64
Q

True or False: Foamed-in-place insulation has a higher insulating value (R-value) than fiberglass batting.

TRUE

FALSE

A

True

65
Q

Which of the following allows a buyer or seller to make informed decisions regarding the property if it has been identified as a site requiring cleanup? (Select one)

Environmental assessment.

Property management solutions.

Risk management plan.

Simple executor plan.

Superfund site.

A

Environmental assessment.

66
Q

Consider the need for real estate professionals to be safe. Carefully read each of the items below and identify those that correctly identify safety protocols to follow. (Select ALL that apply)

When you arrive to a showing, do a quick safety check—note small rooms or potential traps. Ensure there is an exit other than the main entrance, and make sure it is accessible and unlocked. Check the back gates to ensure you can escape the yard.

Ensure all meetings are in daylight hours since bad things never happen during the day

When placing your signs at the property, make sure you are stopping safely. Be aware of other vehicles around you when you are getting in and out of your vehicle.

Remember that only others are susceptible to workplace violence and harm, you personally do NOT need to remain vigilant with respect to your safety.

Plan to check in with someone following a meeting with an unknown client.

A

When you arrive to a showing, do a quick safety check—note small rooms or potential traps. Ensure there is an exit other than the main entrance, and make sure it is accessible and unlocked. Check the back gates to ensure you can escape the yard.

When placing your signs at the property, make sure you are stopping safely. Be aware of other vehicles around you when you are getting in and out of your vehicle.

Plan to check in with someone following a meeting with an unknown client.

67
Q

Yolanda is showing a house to a new client. The client contacted her from an MLS posting, and requested a ride to the house. In this situation, what can Yolanda do to protect herself and remain safe? (Select ALL that apply)

Bring a second person to the showing.

Do NOT inform anyone of the time of the showing.

Meet the client at the house being shown, rather than provide a ride.

Pick up the client at the client’s preferred location.

Turn the cellphone off when meeting with the client.

A

Bring a second person to the showing.

Meet the client at the house being shown, rather than provide a ride.

68
Q

Ursula and her husband Barney are selling their parents’ old property. The old garage has been torn down, but the old house is still standing and is NOT undergoing renovations prior to the sale. During the property inspection, the garage area’s dirt is inspected and found to have a significant amount of grease and oil. Barney notes to the inspector, “oh, that’s just where Dad used to dump all the oil from the cars he worked on. He probably just didn’t dig the hole deep enough that time.”

The property inspector moves to the house and finds the attic is filled with asbestos as insulation and is well-contained and undisturbed. When the inspector goes into the crawlspace with a flashlight (there is no light down there), he discovers the water lines are Poly-B but have copper joints and there is no cracking.

For the above scenario, evaluate each of the following statements, and select those that are TRUE. (Select ALL that apply)

The soil contaminants are NOT an issue, because it’s a residential property.

The soil contaminants are a potential issue.

Asbestos must be removed and replaced before sale of the property.

Asbestos that is NOT being disturbed by renovations (or other movement) is NOT a prohibition to the sale of the property but MUST be disclosed to potential buyers.

Poly-B with copper joints is NOT reason to be concerned because they are in the dark basement.

Poly-B with copper joints may be an elevated leak risk, and a potential buyer should check with their insurance company to see if the building can be insured.

A

The soil contaminants are a potential issue.

Asbestos that is NOT being disturbed by renovations (or other movement) is NOT a prohibition to the sale of the property but MUST be disclosed to potential buyers.

Poly-B with copper joints may be an elevated leak risk, and a potential buyer should check with their insurance company to see if the building can be insured.

69
Q

Reginald is working with a client to build a property on a parcel of land that has recently been zoned for commercial development. The client is looking to secure a permit that confirms that the use of the property complies with the municipalities land use plan and the associated land use bylaws. What type of permit does Reginald’s client need? (Select one)

Building permit.

Development permit.

Improvement authorization permit.

Property construction permit.

Setbacks and easements permit.

A

Development permit.

70
Q

Secondary suites (self-contained units within other units) that do not have a secondary occupancy permit are considered illegal suites. Which of the following reasons are why a property manager should not be involved with illegal secondary suites? (Select ALL that apply)

Conform to current zoning bylaws.

Constructed without proper permits or inspections.

Is in an area where secondary suites are NOT allowed in the land use zoning.

Meets current building codes.

Cannot be legally rented or advertised since a licensed property manager cannot participate in illegal activity

A

Constructed without proper permits or inspections.

Is in an area where secondary suites are NOT allowed in the land use zoning.

Cannot be legally rented or advertised since a licensed property manager cannot participate in illegal activity

71
Q

A municipal land use plan must be diverse and thoughtfully planned to satisfy various needs and benefit the local population as a whole. Which of the following are reasons for the municipalities’ policies on land use (Select ALL that apply)

Aesthetics.

Better education for primary students.

Decrease in overall taxes required for municipality council members.

Economic development.

Environmental sustainability.

Improved prices for property sellers.

Providing adequate infrastructure for properties.

Public health.

Reduction of conflicts between property owners.

A

Aesthetics.

Economic development.

Environmental sustainability.

Providing adequate infrastructure for properties.

Public health.

Reduction of conflicts between property owners.

72
Q

Which of the following residential property types is the most common but does NOT have a shared wall with another unit, is usually the most spacious, and often has a private yard? (Select one)

Apartment.

Detached.

Duplex.

Row.

Semi-detached.

Triplex.

Fourplex.

A

Detached.

73
Q

A brokerage shall keep separate and have available at its registered business office in Alberta or the City of Lloydminster all records for each trade or potential trade.

TRUE

FALSE

A

True

74
Q

Often, conflicts will occur during meetings with clients, with other agents, or in public situations. Some strategies or communication methods can be employed to help prevent conflict, others to resolve conflict. Which of the following methods can be used to prevent conflict? (Select ALL that apply)

Active Listening.

Be dismissive.

Call security if threats of violence are made.

Listen without formulating responses.

Map out next steps, pivot if appropriate.

Multitasking during conversations.

Paraphrase the other party’s viewpoint.

A

Active Listening.

Listen without formulating responses.

Map out next steps, pivot if appropriate.

Paraphrase the other party’s viewpoint.

75
Q

In many situations, minor consumer complaints are NOT required to go through the formal complaint process by contacting RECA. What is the process that allows the consumer, broker, and real estate professional to meet and either negotiate on their own or involve a mediator? (Select one)

Court-Mandated Mediation Meetings (CMMM).

Resolution of Associated Grudge Eradication (RAGE).

Mediated Conflict Resolution Process (MCRP).

Voluntary Association for Mediation (VAM).

Voluntary Broker Resolution Process (VBRP).

A

Voluntary Broker Resolution Process (VBRP).

76
Q

Which of the following outcomes are results of practicing empathy with your clients? (Select ALL that apply)

Assists in building stronger and healthier personal and professional relationships.

Improves understanding of your client’s situations.

The frequency of having to bring a broker in to resolve disagreements between agents and clients will increase.

The more you are able to share in their experience, the more successful you will be in helping them through their situation.

Your relationships will improve to the point that you no longer have conflict.

A

Assists in building stronger and healthier personal and professional relationships.

Improves understanding of your client’s situations.

The more you are able to share in their experience, the more successful you will be in helping them through their situation.

77
Q

True or False: It is much more likely that the seller will receive a more favourable offer for their property when more than one buyer is interested.

TRUE

FALSE

A

True

78
Q

Christopher and Allison are selling their home. As their agent, on the day of listing you receive four separate offers. Two of them have 24-hour consideration clauses, and the other two, while lower, allow more favorable closing conditions (one doesn’t even require an inspection!). As their agent, what actions should you take? (Select ALL that apply)

Discuss the advantages and disadvantages, with Chris and Allison, of informing the other buyers of the offers.

Inform Chris and Allison that they may accept an offer, or counter on any offers.

Pick out the best two offers and present them to Chris and Allison.

Reject all offers and request rebids, now that you know the market is a seller’s market.

Show all offers to the buyers and request rebids before showing the offers to Chris and Allison.

Show all offers to the sellers.

Present offers on the basis of all their terms, conditions, dates, inclusions, and exclusions and not only on price

A

Discuss the advantages and disadvantages, with Chris and Allison, of informing the other buyers of the offers.

Inform Chris and Allison that they may accept an offer, or counter on any offers.

Show all offers to the sellers.

Present offers on the basis of all their terms, conditions, dates, inclusions, and exclusions and not only on price

79
Q

Occasionally, the relationship between an agent and their client breaks down. This most often happens when the client is expecting the agent to perform a certain way, which may differ from what they offer or are capable of. Or, the client’s actions and decisions are making it difficult for the agent to fulfil their obligations to the client. Who is the person to discuss the different options and walk the agent through the process of potentially ending the relationship? (Select one)

A neutral broker from another brokerage.

A neutral arbiter assigned by RECA.

Any other associate from the brokerage.

The agent’s broker.

A

The agent’s broker.

80
Q

In Alberta, there is legislation that encompasses hazards which can be physical, emotional, or mental. These are generally termed occupational health and safety.

True or False: Your brokerage should provide you with information on the OHS guidelines in Alberta and how they are implemented on the job.

TRUE

FALSE

A

True

81
Q

Terrance is a commercial real estate broker who has a client that texts questions that often have difficult to answer responses. For example, a question will be asked that is dependent on multiple variables before an answer can be provided. For clarity and accuracy, what methods of communication could Terrance use that would enable him to quickly gather additional information from the client and provide the best possible response? (Select ALL that apply)

Email.

Face-to-face.

Messaging Tools (e.g., What’s App).

Telephone call

Online advertisement.

Text.

Video Conference.

A

Face-to-face.

Telephone call

Video Conference.

82
Q

According to the Real Estate Act Rules, your brokerage is required to keep relevant documents for a specific period of time. Review each of the statements below. Select the statement that is TRUE. (Select one)

All records, whether electronic or hardcopy must be kept for a period of 5 years.

All records, whether electronic or hardcopy must be kept for a period of 3 years.

Electronic records must be kept for a period of 5 years, whereas hardcopy records must be kept for a period of 3 years.

Electronic records must be kept for a period of 3 years, whereas hardcopy records must be kept for a period of 5 years.

A

All records, whether electronic or hardcopy must be kept for a period of 3 years.

83
Q

Which of the following factors impact ethics and professional conduct? (Select ALL that apply)

Act honestly.

Following legislation, RECA regulations, and brokerage policies and procedures.

Fulfill fiduciary obligations to clients.

Having lunch that includes starches, proteins, carbohydrates and hydration.

Complying with brokerage policies and procedures

Owning a pet.

Taking a yearly vacation.

A

Act honestly.

Following legislation, RECA regulations, and brokerage policies and procedures.

Fulfill fiduciary obligations to clients.

Complying with brokerage policies and procedures

84
Q

Real estate professionals should have both integrity, and be transparent with their clients. Which of the following are examples of transparency. (Select ALL that apply)

Being upfront about your availability.

Declining an offer from a buyer to buy you lunch in order for you provide them insight into your client base.

Disclosing a potential perceived conflict of interest even when it doesn’t exist.

Double checking facts and figures to ensure the offers are accurate.

Explaining to the client the potential costs of various professional services the client might obtain during the transaction.

Sharing with a client questions potential buyers had about the property at the open house.

A

Being upfront about your availability.

Disclosing a potential perceived conflict of interest even when it doesn’t exist.

Explaining to the client the potential costs of various professional services the client might obtain during the transaction.

Sharing with a client questions potential buyers had about the property at the open house.

85
Q

Which of the following active listening techniques involves you explaining how you will follow up or how their comments are going to change your course of action (or not)? (Select one)

Approach the situation with empathy.

Do not multitask.

Listen without formulating responses.

Map out next steps, pivot if appropriate.

Paraphrase.

Use professional language and tone.

Watch your body language.

A

Map out next steps, pivot if appropriate.

86
Q

While grit and resilience are important, they are useless if not partnered with a good state of mind. Which of the following contribute to practicing in a manner that maintains personal health and wellbeing? (Select ALL that apply)

Ask friends and family to check in with you if they are concerned about your mental wellness.

Choose clients whose personalities clash with yours to give you a personal challenge.

Ensuring that you are establishing your business to include things that you enjoy.

Have a can-do attitude by removing all social events.

Set boundaries when necessary.

A

Ask friends and family to check in with you if they are concerned about your mental wellness.

Ensuring that you are establishing your business to include things that you enjoy.

Set boundaries when necessary.

87
Q

During your career as a real estate professional, you are likely to be challenged by clients to make unethical choices. Which of the following is NOT included in the seven-step process for ethical decision making created by philosopher Michael Davis? (Select one)

Identify the problem.

Examine the facts.

Identify relevant factors.

Develop your options.

Calculate monetary value of options.

Test your options.

Make your choice.

Reflect.

A

Calculate monetary value of options.

88
Q

RECA can conduct its own investigations and make any rules it considers appropriate, with certain limitations. Which of the following are the limitations that limit the RECA’s rules? (Select ALL that apply)

RECA’s rules must NOT impact the public.

The rules must NOT contradict the Real Estate Act.

The rules must be ratified by 67% of all licensees.

The rules must NOT contradict the regulations put in place by the government

A

The rules must NOT contradict the Real Estate Act.

The rules must NOT contradict the regulations put in place by the government

89
Q

Judy and William are both agents in the same brokerage. Judy’s friend, Beth, comes into the brokerage and asks for Beth. William greets her at the front desk. Beth is told Judy is out and William asks Beth to describe why she is there. Beth explains she is looking for an agent to work with to sell her home, and Judy, her friend, is a known agent.

Because Judy is out of office, William offers to meet with Beth instead. During their conversation, William has her sign as a client for the next 6 months with a written service agreement. When Judy gets back to the office, she talks with Beth, who is excited to tell her that she’s selling her home.

When Judy thanks her for coming in to see her, Beth notes, “William talked to me while you were gone and had me sign the contract. That means we get to work together, right?”

Judy knows that the brokerage is a designated brokerage and looks at the contract. As expected, Judy finds that it is a sole agency contract between William and Beth. Judy is NOT listed.

Which of the following statements are TRUE? (Select ALL that apply).

Because Beth is Judy’s friend, Judy can actively try to get her friend to break her contract with William to sign with Judy.

Legally, William hasn’t done anything wrong.

Ethically, William may have crossed boundaries of professionalism.

Judy can take the issue to the Broker.

Judy has absolutely no recourse of action and must accept that William is Beth’s real estate agent.

A

Legally, William hasn’t done anything wrong.

Ethically, William may have crossed boundaries of professionalism.

Judy can take the issue to the Broker.

90
Q

A complaint has been made against Jacob to the registrar. The registrar opened an investigation and recommended the issue be brought to a hearing panel. The hearing panel suspended Jacob’s license and imposed a fine of $10,000. Jacob believes the suspension is out of line with the mistake that he readily admitted to, during the investigation. Jacob appealed the hearing panel’s decision, and the appeal panel has reviewed the information.

True or False: While the appeal panel can confirm or throw out the hearing panel’s decision, the appeal panel may NOT change the hearing panel’s decision.

True.
False.

A

True

91
Q

One of the ways RECA fulfills its purposes, particularly those around administering the Real Estate Act, is by creating bylaws. The Real Estate Act states that the Board may make bylaws regarding which of the following? (Select ALL that apply)

Authorizing the board to set up and administer the licensee insurance plan.

Exempting people from the bylaws.

How officers and members of the board and industry councils are elected or appointed.

Management of money received by RECA, the board, or industry councils.

Overseeing education of Alberta citizens and the associated selection, delivery, and distribution of content to the same.

The duties of officers and members of Parliament, and how much they are paid.

What services and materials the board may provide to licensees or the public.

A

Authorizing the board to set up and administer the licensee insurance plan.

Exempting people from the bylaws.

How officers and members of the board and industry councils are elected or appointed.

Management of money received by RECA, the board, or industry councils.

What services and materials the board may provide to licensees or the public.

92
Q

While insurance coverage helps manage the risk of damages, a brokerage may also reduce risk by creating policies and procedures. How do brokerage policies and procedures serve to manage risk? (Select ALL that apply)

Anticipation of situations allows brokerages to put in policies and procedures to prevent the situations from occurring.

Policies and procedures cover all the situations where things can go wrong.

Policies and procedures around communication help keep the broker involved in critical decisions that can mitigate risk.

The trust fund provided by the brokerage spreads the cost of damages and failed contracts across the agents.

A

Anticipation of situations allows brokerages to put in policies and procedures to prevent the situations from occurring.

Policies and procedures around communication help keep the broker involved in critical decisions that can mitigate risk.

93
Q

How does a designated agency relationship with a client prevent conflicts of interest between licensees at the same brokerage? (Select ALL that apply)

Brokerage policies and procedures ensure that one designated agent doesn’t tell the other designated agent anything that might harm their respective client’s interests.

Procedures are established and followed to protect the client’s confidential information.

The brokerage treats the interests of both all clients evenly; information is freely shared between agents of the brokerage to facilitate fair practice of all clients.

The relationship created is a sole agency relationship that prevents the sharing of information across the brokerage.

Sharing of information ensures the best deal or best interest is considered for all parties, regardless of whether they are represented by the brokerage.

A

Brokerage policies and procedures ensure that one designated agent doesn’t tell the other designated agent anything that might harm their respective client’s interests.

Procedures are established and followed to protect the client’s confidential information.

The relationship created is a sole agency relationship that prevents the sharing of information across the brokerage.

94
Q

Gabriella has taken on a new client, Sarai, who is very shy. Sarai brings a friend, Anna, to the meetings to help her better communicate what she wants. During the meetings, Gabriella notices that Anna is often injecting her own opinion over the top of Sarai’s, but Sarai seems reluctant to disagree with her openly. Anna also calls occasionally to give instructions to Gabriella, and once even called to tell Gabriella she should put an offer on a property for Anna.

Which of the following are professional options that Gabriella has to reduce the risk to the transaction? (Select ALL that apply)

Ensure before writing any contract that Sarai’s wants and needs are being addressed, rather than Anna’s wants and needs.

Establish with Anna that Gabriella is Sarai’s agent, and as such all requests and actions must come through Sarai.

Inform Anna that while she is welcome to attend with Sarai, she must NOT talk when Sarai is in the room.

Talk with the broker, and see if the broker has any other ideas on how to reduce risk.

Tell Anna she is NOT welcome to attend with Sarai.

Write all meeting notes down and ask for Sarai to approve in writing any decisions taken.

A

Ensure before writing any contract that Sarai’s wants and needs are being addressed, rather than Anna’s wants and needs.

Establish with Anna that Gabriella is Sarai’s agent, and as such all requests and actions must come through Sarai.

Talk with the broker, and see if the broker has any other ideas on how to reduce risk.

Write all meeting notes down and ask for Sarai to approve in writing any decisions taken.

95
Q

Following a RECA investigation, who receives the written report from the investigators? (Select one)

The broker of record of the brokerage or associate under investigation.

The Minister of Service Alberta.

To the brokerage or associate under investigation.

To the head of the respective industry council.

To the RECA Registrar.

A

To the RECA Registrar.

96
Q

In a contract or an agreement, there are generally two parties. Anyone outside the contract is a third party. Which of the following scenarios would result in third party liability by the licensee? (Select ALL that apply)

A licensee knows the purchase of a home is contingent on the sale of another property but fails to include the contingency in the contract. The purchase falls through, and the client is sued for damages. The purchase fell through because the client’s other property didn’t sell. The client claims the licensee was liable for damages as the licensee knew the financing was contingent and didn’t include it in the contract.

An agent hits another agent’s vehicle backing up in the parking lot. The agent with the damages is third party liable.

Someone brings a claim against a licensee for encroachment of property. The licensee’s personal driveway was recently poured, and it is believed to be on the wrong side of the property line of the licensee. The neighbor is the third party, and the licensee is liable.

Someone brings a claim against the licensee’s client and the client then clamed the damages were caused by the licensee’s actions or omissions.

A

A licensee knows the purchase of a home is contingent on the sale of another property but fails to include the contingency in the contract. The purchase falls through, and the client is sued for damages. The purchase fell through because the client’s other property didn’t sell. The client claims the licensee was liable for damages as the licensee knew the financing was contingent and didn’t include it in the contract.

Someone brings a claim against the licensee’s client and the client then clamed the damages were caused by the licensee’s actions or omissions.

97
Q

For most legislation in Canada, including the Real Estate Act (REA), there are key terms that are defined in the legislation. Which key term in the Real Estate Act is described as “Real property, leasehold property, or a portable dwelling (other than a recreation vehicle)?” (Select one)

Bylaws.

Council.

Industry Council.

Real Estate.

Rules.

A

Real Estate.

98
Q

Not only do licensees have the responsibility of behaving properly themselves, but they are also responsible for supporting the proper function of their profession. Under the Real Estate Act Rules, certain behaviors (unethical, unsafe, illegal, incompetent) should be reported. From the following list, select to whom and what order the actions should be reported? (Select one)

Law enforcement (when appropriate), the person being complained against, then RECA.

RECA first, then the broker, then law enforcement (when appropriate).

The broker first, then RECA or law enforcement (when appropriate).

The broker only. The broker reports anything further to RECA and/or other authorities.

To the person being complained against first, then RECA, then the broker, then law enforcement (when appropriate).

A

The broker first, then RECA or law enforcement (when appropriate).

99
Q

True or False: A conflict of interest can be either real (a real conflict exists) or perceived (the perception of conflict exists).

True.

False.

A

True

100
Q

Following the report of the investigation, if the conduct has been determined to be sufficient for sanction, a disciplinary proceeding can take place. Which of the following disciplinary proceedings may take place if there is enough evidence that the licensee’s conduct deserves sanction? (Select ALL that apply)

A notice of decision is sent to the licensee (and the complainant if there is one).

Registrar imposes an “administrative penalty” the amounts of which are set out in the bylaws.

Registrar sends the findings to an appeals court and the licensee may be compelled to respond to the allegations.

Registrar sends the findings to the Minister of Service Alberta, who determines either to suspend or revoke the license of the licensee.

The Licensee receives a letter, reprimanding the behavior.

A

A notice of decision is sent to the licensee (and the complainant if there is one).

Registrar imposes an “administrative penalty” the amounts of which are set out in the bylaws.

The Licensee receives a letter, reprimanding the behavior.

101
Q

Self-governance (self-regulation) includes the right given by the provincial government, for certain groups to determine which of the following? (Select ALL that apply)

What form the discipline of members should take.

What nationality or decent the members of the group should have.

What qualifications an individual must meet to join the group.

Under what circumstances a member should be disciplined.

Under what circumstances a member should be ejected from the group.

Where the clubhouses are located.

A

What form the discipline of members should take.

What qualifications an individual must meet to join the group.

Under what circumstances a member should be disciplined.

Under what circumstances a member should be ejected from the group.

102
Q

The Real Estate Act can be amended to account for new conditions and provincial needs. Who has the authority to amend the Real Estate Act? (Select one)

The provincial legislature.

The Minister of Service Alberta.

The RECA Board of Directors.

The REA Board.

A

The provincial legislature.

103
Q

Improper conduct results in a conduct review, leading to regulatory or disciplinary actions. In a Court Remedy, the decision maker is a judge. Which of the following are NOT decisions made by a judge. (Select ALL that apply)

Order equitable remedies (what is fair) such as specific performance.

Order the licensee to pay a fine or other penalty to RECA.

Order the licensee to pay damages and costs directly to the person who was harmed.

Order the licensee to take additional training.

Suspend the licensee’s license.

A

Order the licensee to pay a fine or other penalty to RECA.

Order the licensee to take additional training.

Suspend the licensee’s license.

104
Q

Barbara and Doris are working at the same brokerage. It is a designated brokerage. Barbara and Doris commonly eat lunch together and occasionally share funny stories about situations their clients have been in.

They do not and have not ever shared client information or revealed who the clients are. They absolutely do not share information or even stories about current clients.

Barbara is approached by Doris that Doris’s client is interested in a property Barbara has listed. Does a conflict of interest exist? (Select one)

No

Only if the client is informed the designated broker of the seller is in the same brokerage.

Only if Barbara has shared a story that is similar to something that is happening with Doris’s current client.

Yes

Who pays for lunch?

A

No

105
Q

A concept under common law that presumes all real estate professionals registered with the brokerage share information and advice and have knowledge of confidential information concerning all the clients of the brokerage.

Agency Relationships

Attributed Knowledge

Designated Agency

Transaction Brokerages

A

Attributed Knowledge

106
Q

Under which circumstances can the registrar freeze a licensee’s assets? (Select ALL that apply)

If it looks as though the licensee might move assets around and hide them (including shifting money between accounts or account holders) to prevent an injured party from getting anything back.

There are connected criminal proceedings, or civil proceedings for non-compliance with the Real Estate Act, which are based in the business of a licensee.

When the registrar has appointed someone to investigate a brokerage and instructed a brokerage staff member to produce documents related to the investigation.

Where the amount in the licensee’s trust account is less than what the registrar thinks the licensee might owe.

A

If it looks as though the licensee might move assets around and hide them (including shifting money between accounts or account holders) to prevent an injured party from getting anything back.

There are connected criminal proceedings, or civil proceedings for non-compliance with the Real Estate Act, which are based in the business of a licensee.

Where the amount in the licensee’s trust account is less than what the registrar thinks the licensee might owe.

107
Q

Which of the following actions would reduce risk or be a risk-reduction practice? (Select ALL that apply)

Amending documentation.

Securing insurance.

Submitting paperwork on time.

Using checklists to ensure all actions are taken.

Using your personal vehicle to carry clients to and from appointments.

When unsure make a quick call to the broker.

A

Securing insurance.

Submitting paperwork on time.

Using checklists to ensure all actions are taken.

When unsure make a quick call to the broker.

108
Q

As a real estate professional, you are not authorized to perform certain activities that require a mortgage broker. From the following scenarios, determine which of them either require you to refer the client or contact to a mortgage professional. (Select ALL that apply)

A private lender offers you a fee to connect them with buyers seeking specific rates and terms.

Client asks you to offer advice on mortgage terms and rates.

Client asks you to act as an intermediary between them and their lender.

Client requests you provide an estimate of what the value of their home would be in the market.

Client asks you what the latest Prime Rate is in the market.

Lender asks you to advertise on their behalf or to seek out potential borrowers.

Your client asks you to investigate several banks and what their lending terms are.

A

A private lender offers you a fee to connect them with buyers seeking specific rates and terms.

Client asks you to offer advice on mortgage terms and rates.

Client asks you to act as an intermediary between them and their lender.

Lender asks you to advertise on their behalf or to seek out potential borrowers.

Your client asks you to investigate several banks and what their lending terms are.

109
Q

The Real Estate Act requires anyone trading in real estate as a real estate broker, dealing as a mortgage broker, acting as a property manager, or acting as a real estate appraiser to have the authorization to carry out these activities. In each industry sector, the preliminary factors in determining whether a person’s activities require a license are which of the following? (Select ALL that apply)

Whether the person is being compensated for undertaking these activities or providing these services.

Whether the person is performing a real estate, mortgage, property management, or appraisal activity or service for someone else.

Whether the person is trading in real estate as the buyer or seller.

Whether the person works for a brokerage, regardless of whether or not they trade in real estate.

A

Whether the person is being compensated for undertaking these activities or providing these services.

Whether the person is performing a real estate, mortgage, property management, or appraisal activity or service for someone else.

110
Q

Oliver is working with a seller, Grayson, to list his home. This is Oliver’s first home listing (Grayson knows this, as he’s his neighbour). Oliver has collected all home specifications and is in the process of creating the listing on the MLS®. He asks Grayson if he has pictures of the home to provide. Grayson stops for a moment and asks the question, “Does having great pictures and a nice interior actually make a difference in the selling price of the home?” Oliver explains that the real estate education he completed explained that studies have shown that home prices can be 1%-5% higher with nicely shown and staged homes. Grayson asks Oliver if he wouldn’t mind taking some pictures of the home. Oliver knows he’s a lousy photographer. What should Oliver suggest to Grayson? (Select the BEST response).

Oliver needs to tell Grayson the market is really “hot right now” and that pictures and staging won’t matter.

Oliver should accept the challenge anyway, and offer to take the pictures and charge a nominal fee to Grayson for the pictures he takes. He should tell Grayson to “look up staging” on the internet to learn how to do it.

Oliver should suggest that Grayson look into professional photography solutions along with staging. If Oliver knows 3 or more companies and can vouch for their quality and experience, Oliver can provide that information to Grayson.

Oliver should suggest that Grayson look into professional photography solutions along with staging. Oliver quickly should browse the internet to find local options and make a recommendation based on the number of stars in the reviews.

A

Oliver should suggest that Grayson look into professional photography solutions along with staging. If Oliver knows 3 or more companies and can vouch for their quality and experience, Oliver can provide that information to Grayson.

111
Q

Which of the following are within the scope of a Condominium Management Associate license? (Select ALL that apply)

Blocking access of tenants who violate terms of the corporation.

Collecting, holding or disbursing, or attempting to collect, hold or disburse, contributions levied by the condominium corporation or other amounts levied by or due to the corporation under the Condominium Property Act.

Enforcing the bylaws or rules of the corporation.

Negotiating or entering into contracts on behalf of the corporation.

Representing commercial businesses located adjacent to the condominium property.

Supervising employees or contractors hired or engaged by the corporation.

A

Collecting, holding or disbursing, or attempting to collect, hold or disburse, contributions levied by the condominium corporation or other amounts levied by or due to the corporation under the Condominium Property Act.

Enforcing the bylaws or rules of the corporation.

Negotiating or entering into contracts on behalf of the corporation.

Supervising employees or contractors hired or engaged by the corporation.

112
Q

While home inspectors can get home inspector training from multiple locations, a home inspection business must have a license under which of the following? (Select one)

Canada Mortgage and Housing Corporation (CMHC).

Consumer Protection Act from the Government of Alberta.

Inspection Certification Associates (ICA).

National Home Inspector Certification Council (NHICC).

A

Consumer Protection Act from the Government of Alberta.

113
Q

Which of the following definitions correctly describes monthly condominium contributions? (Select one)

The cost of each unit’s contributions to maintain the board’s salaries.

The difference in maintenance costs between a larger unit and smaller unit in the same condominium property.

The proportion of a unit’s size in relation to the whole condominium property, divided by 12 and expressed as a percentage.

The portion of the condominium corporation’s annual budget assigned to the unit owner on a monthly basis.

A

The portion of the condominium corporation’s annual budget assigned to the unit owner on a monthly basis.

114
Q

When the condominium plan is registered with Alberta Land Titles, which of the following are TRUE? (Select ALL that apply)

A corporation is formed in which members are the owners of the units.

A corporation is formed in which a single member owns all the units (known as the condominium tzar), and all condominium owners legally must provide fealty to the condominium tzar.

The condominium corporation will also receive its legal name, for example, Condominium Corporation No. 8810776.

The members of a condominium corporation must select a legal name, such as Carl’s Condo’s for Crewmembers.

A

A corporation is formed in which members are the owners of the units.

The condominium corporation will also receive its legal name, for example, Condominium Corporation No. 8810776.

115
Q

What does the board delegate to a condominium property management company? (Select one)

The use and distribution of all funds associated with the condominium properties.

Some of the board’s management and fiscal responsibilities.

Unit ownership.

Common property ownership.

A

Some of the board’s management and fiscal responsibilities.

116
Q

Besides a title search on condominium properties, it is also possible to search for what is known as the condominium additional plan sheet. This report will include which of the following information? (Select ALL that apply)

Any changes to the bylaws or directors.

Any registrations made against the condominium plan.

Common property descriptions and plats.

The condominium corporation identification.

The condominium corporation’s street address.

Unit owner’s names

A

Any changes to the bylaws or directors.

Any registrations made against the condominium plan.

The condominium corporation identification.

The condominium corporation’s street address.

117
Q

Jimmy purchased a condominium unit without an estoppel certificate.

Jimmy would be liable for any arrears that are owing for the unit he purchased.

TRUE

FALSE

A

True

118
Q

Which of the following justifies a brokerage’s need to collect seller/landlord and buyer/tenant information during a transaction? (Select one)

Collecting the data prevents the need for client confidentiality.

It is a requirement of the Canada Mortgage and Housing Corporation (CMHC) to establish identities of all parties in a lease/tenant scenario.

It minimizes the possibility of mortgage fraud.

Minister of Service Alberta requires the information for brokerage licensing.

A

It minimizes the possibility of mortgage fraud.

119
Q

In a mortgage who is typically referred to as “the buyer” or individual who is obtaining the loan? (Select one)

Mortgagee.

Mortgagor.

Primary title holder.

Writ of Guarantor.

A

Mortgagor.

120
Q

True or False: Money laundering is a legal process used to disguise the source of money or assets that come from illegal means.

TRUE

FALSE

A

False

121
Q

Which of the following best describes financial risk? (Select one)

The cost of doing business.

The factors associated with a credit score.

The financial factors involved in assigning a value to a credit applicant.

The possibility of losing money as a result of default, loss of income, or costs outweighing profits.

The probability of loss due to economic forces outside of the control of the lender.

A

The possibility of losing money as a result of default, loss of income, or costs outweighing profits.

122
Q

Riley has a client listing on MLS® that is generating a lot of calls about the property. One of the callers asks Riley if he would be willing to represent him as the buyer for the property, and suggests that by doing so, Riley should be extra grateful for the additional commission. Riley appreciates the idea of making more money, but also understands that he needs to talk with the seller client before he can proceed with a transaction. Which of the following would reduce the professional risk to Riley in this situation? (Select ALL that apply)

Riley could suggest the buyer work out a deal with him “under the table” to secure the information the buyer wants.

Riley could suggest another agent from the brokerage to the buyer to avoid any conflict of interest with his current client.

Riley must allow the seller to determine if Riley representing both parties in the deal is acceptable.

Riley should inform the buyer that if he represents both parties in the deal, he cannot offer any details about the seller or offer advice or counsel as a regular agent could, due to the need to remain impartial.

Riley should suggest to the seller that this type of deal will result in a higher sales price for the seller.

A

Riley could suggest another agent from the brokerage to the buyer to avoid any conflict of interest with his current client.

Riley must allow the seller to determine if Riley representing both parties in the deal is acceptable.

Riley should inform the buyer that if he represents both parties in the deal, he cannot offer any details about the seller or offer advice or counsel as a regular agent could, due to the need to remain impartial.

123
Q

The title of the land contains all registered interests. What does the priority order of registered mortgages mean? (Select one)

The claim upon the chattel of the property in the event of default.

The order in which the lenders will be paid upon the sale of property.

The higher the order, the greater percentage of property tax the mortgage provider is required to pay.

The higher the order, the more the mortgage provider assumes loss in the event of default.

A

The order in which the lenders will be paid upon the sale of property.

124
Q

Which of the following is NOT a method to reduce financial risk? (Select one)

Accept a job in an employment field with low turnover.

Check the title for liens or encumbrances before using the property as collateral for a loan.

Mortgage a property with a loan that is less than 80% of the property’s true value.

Save money in a savings account.

Secure multiple loans from lenders who do not report to credit agencies.

Secure a pre-approval for a loan amount.

A

Secure multiple loans from lenders who do not report to credit agencies.

125
Q

Which of the following is a common method for commercial real estate to be used in money laundering schemes? (Select one)

Commercial real estate has higher prices, enabling money launderers to purchase properties of greater value.

Financing for commercial real estate requires additional checks on credit worthiness.

Rentals or leases are paid in all cash sums, often above market rates, to help prevent the landlord or property owner from investigating the source of the funds.

Rentals or leases are used as a base of operations for cleaning money.

A

Rentals or leases are paid in all cash sums, often above market rates, to help prevent the landlord or property owner from investigating the source of the funds.

126
Q

Which of the following activities are considered to be “dealing in mortgages” and require a mortgage brokering license? (Select ALL that apply)

Assisting a client in choosing which mortgage product and lender is best for them.

Collecting client information on behalf of a mortgage lender to secure financing.

Contacting a mortgage lender to get a rate sheet.

Contacting mortgage lenders to secure financing on behalf of a client.

Referring a client to a mortgage broker.

A

Assisting a client in choosing which mortgage product and lender is best for them.

Collecting client information on behalf of a mortgage lender to secure financing.

Contacting mortgage lenders to secure financing on behalf of a client.

127
Q

Some mortgages do not qualify for mortgage insurance. These mortgages require the lender to take on additional risk and are called uninsurable mortgages. Which of the following are characteristics of an uninsurable mortgage? (Select ALL that apply)

A mortgage that is considered to be lent as an auxiliary product.

A mortgage with a 30-year amortization.

A mortgage with a minimum down payment of 20%.

An investment property.

Any mortgage loan on a purchase over one million dollars.

The lowest interest rates of all mortgage products.

A

A mortgage that is considered to be lent as an auxiliary product.

A mortgage with a 30-year amortization.

An investment property.

Any mortgage loan on a purchase over one million dollars.

128
Q

Which of the following mortgage types is a mortgage that is funded at the completion of the new home where the builder does not expect any funds, outside of an initial deposit at the contract stage, until possession of the new home is taken? (Select one)

Build/Completion mortgage.

Construction draw mortgage.

Crashcart mortgage.

Home Equity Line of Credit.

Residential mortgage.

A

Build/Completion mortgage.

129
Q

Which of the following are ways real estate professionals can minimize or suppress mortgage fraud? (Select ALL that apply)

Always ask your client where the referral came from and how they obtained your information.

Avoid meeting clients face to face.

Discourage the seller or investor from having you see or inspect the property.

Encourage consumers to hire a professional to facilitate their purchase.

If a property that is being listed and sold is occupied by a tenant, the real estate professional must collect information from all parties.

Leave signature lines or other important areas on a loan application blank.

Order, review, and verify all land title documents and mortgage statements to confirm ownership before listing or buying properties.

Read, study, and be aware of all mortgage fraud types and transactions.

Tune into Fraud Awareness Month with RECA and other provincial and national governing or licensing bodies.

Verify all documents and fact check your clients’ information.

A

Always ask your client where the referral came from and how they obtained your information.

Encourage consumers to hire a professional to facilitate their purchase.

If a property that is being listed and sold is occupied by a tenant, the real estate professional must collect information from all parties.

Order, review, and verify all land title documents and mortgage statements to confirm ownership before listing or buying properties.

Read, study, and be aware of all mortgage fraud types and transactions.

Tune into Fraud Awareness Month with RECA and other provincial and national governing or licensing bodies.

Verify all documents and fact check your clients’ information.

130
Q

Which of the following is a federal act that imposes requirements on how interest is described and calculated in loan and mortgage documents? (Select one)

The Interest Act.

Service Alberta.

The Law of Property Act.

The National Housing Act.

A

The Interest Act.

131
Q

Which of the following broad categories of fraud is generally carried out by the borrower misrepresenting facts of their financial situation to gain a mortgage approval, resulting in the fraudulent activities enabling the buyer to secure the property for their own use? (Select one)

Fraud for homelessness.

Fraud for housing.

Fraud for private enterprise.

Fraud for profit.

A

Fraud for housing.

132
Q

True or False: Foreclosure proceedings vary from province to province.

TRUE

FALSE

A

True

133
Q

In which type of ownership, if one of the owners should die, do the remaining owners retain the full ownership of the property by survivorship at that death? (Select One)

Conjoined ownership.

Investor deemed.

Joint tenants.

Survival ownership.

Tenants in common.

A

Joint tenants.

134
Q

True or False: A client has the right to prohibiting their agent from disclosing information that is not required by law to be disclosed.

TRUE

FALSE

A

True

135
Q

True or False: Clients who are informed of conflicts of interest do NOT need to be given the opportunity to obtain independent legal advice concerning the conflict.

TRUE

FALSE

A

False

136
Q

Which of the following methods to end or discharge a contract occurs when some element, or some intervening event or circumstance that neither party anticipated, and which is beyond the control or consent of either, makes it impossible to complete the transaction? (Select one)

Breach.

Frustration.

Operation of law.

Performance.

Mutual agreement.

A

Frustration.

137
Q

Which of the following are components to maintaining proper documentation? (Select ALL that apply)

Immediately dispose of contracts that had conditions unmet.

Keep notes and documents relating to a transaction together.

Keep records of all signed documentation

Maintain electronic records in an accessible format.

Maintain electronic records in difficult to access methods and locations.

A

Keep notes and documents relating to a transaction together.

Keep records of all signed documentation

Maintain electronic records in an accessible format.

138
Q

Recently, you represented a client who, when attempting to close a real estate transaction, disputed with the other party about verbally agreed-upon actions that were to be taken before the close of the deal. The deal ended up collapsing, and both parties were very angry at the situation. What could you advise the next client to help prevent such a breakdown? (Select ALL that apply)

Any changes to the terms of the contract require written amendment signed by both parties.

Any stipulations or agreements must be in writing in the contract to be enforceable.

Negotiations should occur prior to the contract signing.

Negotiating verbally is acceptable as long as both parties are in complete alignment of what is being agreed upon.

Negotiating verbally is acceptable as long as the parties agree to the requirements (of what was negotiated) in a written and signed document.

A

Any changes to the terms of the contract require written amendment signed by both parties.

Any stipulations or agreements must be in writing in the contract to be enforceable.

Negotiations should occur prior to the contract signing.

Negotiating verbally is acceptable as long as the parties agree to the requirements (of what was negotiated) in a written and signed document.

139
Q

True or False: Real estate professionals have an obligation to ensure final signed copies of all agreements are provided to all parties to a transaction in a timely manner.

TRUE

FALSE

A

True

140
Q

Occasionally during contract negotiations, differences and disputes arise that put both the contract and the client at risk. What steps can be taken to advise clients and help prevent disputes and differences from resulting in legal action? (Select ALL that apply)

Encourage the client to have the contract reviewed by a lawyer prior to signing.

Include additional clauses written into the contract terms which allow for a specific set of dispute resolution processes to be employed if the parties to the contract have a disagreement pertaining to specific terms, obligations, or entitlements.

Keep the contract time short to prevent disputes from arising.

Use vague and unspecific language in the contract, to give maximum flexibility in meeting requirements.

A

Encourage the client to have the contract reviewed by a lawyer prior to signing.

Include additional clauses written into the contract terms which allow for a specific set of dispute resolution processes to be employed if the parties to the contract have a disagreement pertaining to specific terms, obligations, or entitlements.

141
Q

True or False: Zoning classifications are guidelines only. (e.g. retail space can be built in residential classifications, or vice-versa).

TRUE

FALSE

A

False

142
Q

The Real Estate Act Rules define a conflict of interest as “a real or apparent incompatibility between an industry member’s interests and the interests of the client or potential client.”

Real estate professionals cannot provide services to a client or potential client if a conflict of interest exists unless what condition is met? (Select one)

The client is informed of the conflict verbally or in writing.

The client provides their informed consent in writing.

The conflict of interest is disclosed to the brokerage.

The conflict of interest is in the Real Estate Act exclusion list.

A

The client provides their informed consent in writing.

143
Q

Written communication, documentation, and record-keeping are fundamental to the work of a real estate professional. In situations where an important file note is being made, a best practice is to send a brief written correspondence to the client outlining the issue or discussion and any advice given. In which situation does this apply? (Select one)

Actions.

Advice.

Conversations.

Individual practice.

All of the above.

None of the above.

A

All of the above.

144
Q

Jackie is representing a buyer, Henrietta, who is very much interested in a residential property that is located near her parent’s home. Henrietta makes an offer that is accepted, but there is a condition that requires an $8,000 cash deposit. Henrietta has $6,000 in cash, and needed to come up with another $2,000 by tomorrow. Henrietta decides to sell an ATV to make up the difference. She sells the ATV to her neighbor who provides a cheque to her, which she deposits. While at the bank Henrietta attempts to withdraw the $8,000, but she discovers that the funds for the $2,000 cheque have not cleared yet. Henrietta knows the deal is in jeopardy and approaches Jackie for advice.

Knowing that she must provide the best advice she can on behalf of her client, Jackie should provide which of the following as advice to the buyer? (Select TWO)

Provide the balance of the money in $6,000, and pretend it was a misunderstanding when the amount doesn’t match. Promise the money the next day and see if the seller agrees.

If the deposit is not paid or delivered on time, or in the manner specified, the seller has the right to void the contract by notice in writing.

Henrietta can explain the situation to the seller, and ask for an amendment to change the terms of the deposit, which must be agreed upon in writing.

Write a cheque to the seller, and hope the cheque doesn’t clear before the ATV cheque does.

A

If the deposit is not paid or delivered on time, or in the manner specified, the seller has the right to void the contract by notice in writing.

Henrietta can explain the situation to the seller, and ask for an amendment to change the terms of the deposit, which must be agreed upon in writing.

145
Q

Which of the following remedies for a breach of contract is only available if there is evidence an actual loss occurred due to the breach? (Select one)

Injunction.

Monetary damages.

Negotiation.

Quantum meruit.

Rescission.

Specific performance.

A

Monetary damages.

146
Q

In commercial transactions, which of the following financial statements may be required by the buyer from the seller? (Select ALL that apply)

Annual financial statements.

Balance sheets.

Compilation engagements.

Income (profit and loss) statements.

Monthly financial statements.

A

Annual financial statements.

Balance sheets.

Compilation engagements.

Income (profit and loss) statements.

Monthly financial statements.

147
Q

Jacob is reviewing a title of a property he is interested in purchasing. He notices there are three incumbrances on the title, including two mortgages and a lien. The listing on the title is as follows: Lien, Mortgage from Bank of the Mountains, and then a mortgage from Lenders4U. What does this indicate? (Select one)

At the sale of the home, all three encumbrances have equal claim on the proceeds, and will be paid evenly until all funds are distributed.

At the sale of the home, the Lenders4U is paid first, then Bank of the Mountains, then the lien.

At the sale of the home, the lien must be paid off before the mortgages have claim on the proceeds. Mortgage from Bank of the Mountains must be paid before Mortgage from Lenders4U.

It only indicates there are three encumbrances. If the sale of the property doesn’t cover the amounts of the encumbrances, the three items must go to court and have a judge issue percentage funds.

A

At the sale of the home, the lien must be paid off before the mortgages have claim on the proceeds. Mortgage from Bank of the Mountains must be paid before Mortgage from Lenders4U.

148
Q

Where are the terms of trust negotiated and codified by the buyer/tenant, seller/landlord, and the trustee? (Select one)

In a separate “trust contract” provided by Service Alberta.

In a separate “trust contract” provided by the Real Estate Council of Alberta (RECA).

In the brokerages written service agreement (WSA) with the seller.

In the brokerage’s written service agreement (WSA) with the buyer.

In the real estate contract signed by the parties.

A

In the real estate contract signed by the parties.

149
Q

According to the Electronic Transactions Act (ETA), electronic records and transactions have the same effect and enforceability as traditional paper-based transactions if conditions are met.

True or False: Service agreements must be signed in hardcopy only (electronic records may only be created after the real signature is applied.)

TRUE

FALSE

A

FALSE

150
Q

Which of the following are best practices when working with contracts? (Select ALL that apply)

Encouraging clients to quickly review contract language with assurances that any misunderstandings will be verbally corrected with the other party.

If the parties come to an agreement after the date of acceptance has expired, it is best practice to restart negotiations by re-writing the agreement.

Including provisions in the contract language that indemnifies your brokerage for any missing content requested by the buyer or seller.

Make sure buyers are fully informed and understand the concept of restrictive covenant and are provided with a copy of specific encumbrances so that they are able to read and understand it before agreeing to be bound by its terms.

Real estate professionals representing buyers who will not be waiving conditions providing a formal notice of the non-waiver to the seller.

The use of written service agreements.

A

If the parties come to an agreement after the date of acceptance has expired, it is best practice to restart negotiations by re-writing the agreement.

Make sure buyers are fully informed and understand the concept of restrictive covenant and are provided with a copy of specific encumbrances so that they are able to read and understand it before agreeing to be bound by its terms.

Real estate professionals representing buyers who will not be waiving conditions providing a formal notice of the non-waiver to the seller.

The use of written service agreements.

151
Q

Which type of contract is an agreement where a client agrees to hire the brokerage to act on their behalf in a real estate transaction? (Select one)

Brokerage or associate agreements.

Brokerage service agreements.

Fee Simple contract.

Real estate purchase or lease contracts.

A

Brokerage service agreements.

152
Q

When reviewing documentation, which of the following techniques fosters accuracy and clarity? (Select ALL that apply)

Confirm attachment of all supporting documentation.

Ensure all signatures and initials are in place.

Ensure contracts contain proper dates.

Sign and initial agreements on the client’s behalf.

Ensure copies of all documents are given to all parties to the contract.

Ensure the terms and conditions of the offer reflect the client’s understanding and are clear to other parties.

When unsure of legal requirements, instruct the clients to do their own research rather than seek legal counsel, to save money.

Review the contract with the client carefully, so the client fully understands all terms.

A

Confirm attachment of all supporting documentation.

Ensure all signatures and initials are in place.

Ensure contracts contain proper dates.

Ensure copies of all documents are given to all parties to the contract.

Ensure the terms and conditions of the offer reflect the client’s understanding and are clear to other parties.

Review the contract with the client carefully, so the client fully understands all terms.